colleen's photo is 9 inches long and 7 inches wide. is it larger or smaller than ali's photo? explain how you know.

Answers

Answer 1

By calculations, Colleen's photo is smaller than Ali's photo

Determining if Colleen's photo larger or smaller than Ali's photo?

From the question, we have the following parameters that can be used in our computation:

Area of Ali's photo = 91 square inches.

For Colleen's photo, we have

9 inches by 7 inches

This means that

Area of Colleen's photo = 9 * 7 square inches.

Evaluate

Area of Colleen's photo = 63 square inches.

63 square inches is lesser than 91 square inches

This means that Colleen's photo is smaller than Ali's photo

Read mroe about expression at

https://brainly.com/question/15775046

#SPJ1


Related Questions

In a circle, an angle measuring 2.2radians intercepts an are of length 11.9.Find the radius of the circle to the nearest
10th.

Answers

The radius of the circle to the nearest 10th is 5.4

Showing how to calculate radius

The formula for calculating the length of an arc of a circle is:

length of arc = radius x angle in radians

l = rθ

where

r = radius of the circle

l = length of arc

θ = angle in radians

From the question, we are given:

length of arc = 11.9

angle in radians (θ) = 2.2radians

The we can plug in the values

11.9 = r x 2.2

make r the subject of the formula

r = 11.9/2.2

r = 5.41 (to 2 decimal places)

Learn more about radius here:

https://brainly.com/question/27696929

#SPJ1

 Part C
The rectangular sides of the treasure box will be cut from wooden planks
5
9 feet long and foot wide. How many planks will Mr. Penny need so
9
16
that his 18 students can each construct one treasure box?

Answers

Mr. Penny will require a total of 20 square feet of wooden planks for all 18 students to construct their treasure boxes.

To determine the number of planks required, we need to calculate the total amount of wood needed for all 18 students' treasure boxes.

Each treasure box has two identical rectangular sides.

Each side is cut from a wooden plank that is 5/9 feet long and 1 foot wide.

Therefore, the area of each side is [tex](5/9) \times 1 = 5/9[/tex] square feet.

Since there are two identical sides for each treasure box, the total area of wood needed for one treasure box is [tex](5/9) \times 2 = 10/9[/tex] square feet.

To find the total wood needed for 18 students' treasure boxes, we multiply the area per treasure box by the number of treasure boxes:

Total wood needed [tex]= (10/9) \times 18 = 20[/tex] square feet.

So, Mr. Penny will require a total of 20 square feet of wooden planks for all 18 students to construct their treasure boxes.

For similar question on rectangular sides.

https://brainly.com/question/29001951

#SPJ11

Question: What is the number of planks required for Mr. Penny's 18 students to each construct one treasure box if the rectangular sides of the treasure box will be cut from wooden planks that are 5/9 feet long and 1 foot wide?

Help guys asap i need correct answers only!! find the volume of the cylinder. find the volume of a cylinder with the same radius and double the height.



the volume of the cylinder in^3?



the volume of with the same radius and double the height is

Answers

To find the volume of the cylinder, we need to use the formula:

V = πr^2h

where V is the volume, r is the radius, and h is the height.

If we have a cylinder with radius r and height h, and another cylinder with the same radius r but double the height (2h), the volume of the second cylinder is:

V' = πr^2(2h) = 2πr^2h

So, to answer the questions:

The volume of the cylinder is V = π(5 cm)^2(8 cm) = 100π cubic cm, which is approximately 314.16 cubic cm rounded to two decimal places.

The volume of the cylinder with the same radius and double the height is V' = 2π(5 cm)^2(8 cm) = 200π cubic cm, which is approximately 628.32 cubic cm rounded to two decimal place

Find an angle in each quadrant with a common reference angle with 306°, from 0°≤θ<360°

Answers

The angle in each quadrant with a common reference angle with 306° are

Quadrant 1 is 126°Quadrant 2 is 54°Quadrant 3 is  306°Quadrant 4 is 234°

To find angles in each quadrant with a common reference angle with 306°, we first need to determine the reference angle for 306°.

Reference angle is the acute angle between the terminal side of an angle and the x-axis. We can find the reference angle for any angle θ by subtracting the nearest multiple of 180° from θ and taking the absolute value of the result. In this case:

|306° - 180°| = 126°

So, the reference angle for 306° is 126°.

Now, we can find an angle in each quadrant with a common reference angle of 126°:

1st quadrant: The angle with a reference angle of 126° in the 1st quadrant is simply 126°.

2nd quadrant: To find the angle with a reference angle of 126° in the 2nd quadrant, we need to subtract the reference angle from 180° (since all angles in the 2nd quadrant are between 90° and 180°).

180° - 126° = 54°

So, an angle with a reference angle of 126° in the 2nd quadrant is 54°.

3rd quadrant: To find the angle with a reference angle of 126° in the 3rd quadrant, we need to subtract the reference angle from 180° and then add 180° (since all angles in the 3rd quadrant are between 180° and 270°).

180° + 126° = 306°

So, an angle with a reference angle of 126° in the 3rd quadrant is 306°.

4th quadrant: To find the angle with a reference angle of 126° in the 4th quadrant, we need to subtract the reference angle from 360° (since all angles in the 4th quadrant are between 270° and 360°).

360° - 126° = 234°

So, an angle with a reference angle of 126° in the 4th quadrant is 234°.

To know more about angles visit:

brainly.com/question/28451077

#SPJ11

write a math sequence for this problem! lilah and her little brother max went to the beach they dug 290 seashells and 645 rocks how many rocks and seashells did max and lilah collect in all?

Answers

Answer:

935 Rocks and shells

Step-by-step explanation:

To find the total number of rocks and seashells collected by Max and Lilah, we can use the addition operation. Let S be the number of seashells and R be the number of rocks. Then, the math sequence for this problem is:

S + R = Total

Substituting the given values, we get: 290 + 645 = Total

Simplifying the right-hand side, we get:

935 = Total

Therefore, Max and Lilah collected a total of 935 rocks and seashells in all.

To find the total number of rocks and seashells collected by Lilah and Max, we simply need to add the number of seashells and rocks they each collected. Let S represent the number of seashells and R represent the number of rocks. Then, the equation is:

S + R = 290 + 645

Simplifying this expression, we get:

S + R = 935

935 rocks and seashells.

The ratios of successive numbers in the Fibonacci sequence eventually get closer to which number?


a.


1. 61


c.


2. 3


b.


1. 46


d.


1

Answers

The ratios of successive numbers in the Fibonacci sequence eventually get closer to a. 1.61


In mathematics, the Fibonacci sequence is a sequence in which each number is the sum of the two preceding ones. Numbers that are part of the Fibonacci sequence are known as Fibonacci numbers, commonly denoted Fn . The sequence commonly starts from 0 and 1, although some authors start the sequence from 1 and 1 or sometimes (as did Fibonacci) from 1 and 2.

The Fibonacci sequence is a series of numbers where each number is the sum of the two preceding ones, usually starting with 0 and 1 (0, 1, 1, 2, 3, 5, 8, 13, ...). When you take the ratio of successive numbers in the sequence (e.g., 5/3 or 8/5), it converges to approximately 1.618, also known as the Golden Ratio or Phi.

The closest option in your list is 1.61, which is option (a).

Learn more about "Fibonacci sequence": https://brainly.com/question/16934596

#SPJ11

You pick a card at random. (1234)
What is P(greater than 1 or divisor of 2)?

Answers

The probability of picking a number greater than 1 or a divisor of 2 is 3/4.

Calculating the probability P(greater than 1 or divisor of 2)?

The numbers greater than 1 or divisor of 2 in the set {1, 2, 3, 4} are {2, 3, 4}.

Therefore, the probability of picking a number greater than 1 or a divisor of 2 is:

P(greater than 1 or divisor of 2) = P(2 or 3 or 4) = P(2) + P(3) + P(4)

Since there are four equally likely outcomes, the probability of each individual outcome is 1/4. Thus, we have:

P(greater than 1 or divisor of 2) = 1/4 + 1/4 + 1/4 = 3/4

Read more about probability at

https://brainly.com/question/251701

#SPJ1

find the missing side length

Answers

Answer:

Step-by-step explanation:

[tex]x^{2}[/tex]=a²+b²

x²=6²+2.5²

x²=36+6.25

x²=42.25

√x=√42.25

x=6.5

Answer:

6.5

Step-by-step explanation:

6^2 + 2.5^2 = c^2

•36 + 6.25 = c^2

•42.25 = c^2

• find square root of 42.25 = 6.5

• 6.5 = C

Luis created a spreadsheet of his expenses for three months. Which of Luis's expenses are variable expenses


utility bill


Expenses Jan Feb Mar


rent


$1,250,00 $1,250,00 $1,250,00


$124. 11 $108. 72 $121. 69


car loan payment $384. 00 3384,00 $384. 00


Insurance payment 397. 18 597. 18 $97. 18


groceries


$315,43 $367. 25 $341. 04


clothing


$72. 18 $152. 74 $0. 00


fuel


$108. 71 $117. 46 $127. 34

Answers

Variable expenses are expenses that fluctuate from month to month, and are typically not a fixed amount. Examples of variable expenses include groceries, fuel, clothing, and entertainment. These expenses can be influenced by various factors such as personal choices, seasonality, and external events.

In Luis's expenses, the following expenses are variable expenses:

Groceries: The amount spent on groceries changes from month to month depending on the types of food Luis purchases and the quantity he buys.Clothing: This expense is variable because Luis only spent money on clothing in January and February, and did not spend anything on clothing in March.Fuel: The amount spent on fuel changes from month to month depending on how often Luis drives and the price of gasoline.

On the other hand, the following expenses are fixed expenses:

Rent: This expense is fixed because Luis pays the same amount for rent every month.Car loan payment: This expense is fixed because Luis is required to pay the same amount for his car loan every month.Insurance payment: This expense is fixed because Luis is required to pay the same amount for his insurance every month.Utility bill: The utility bill could be either a variable or fixed expense, depending on the type of utility. For example, if the utility bill is for electricity, it may be a variable expense because the amount of electricity used can fluctuate from month to month. However, if the utility bill is for a fixed service such as internet, it would be a fixed expense.

To learn more about “utility” refer to the https://brainly.com/question/14729557

#SPJ11

Answer:

groccieries is the answer for plato 2023

A taho vendor having lost control of his cart down a slight hill runs after it in an attempt to keep it from running into a concrete wall however he did not get there in time and the 100 kg cart crashes assuming that in its downhill run the cart got a final velocity of 2m/s and that the impact stopped the cart in 0.15s, (a) determine the change in the cart's momentum (b) estimate the average force that the wall exerts on the cart (neglecting the angle of the hill) (c) determine the direction of the impulse that acted on the cart

Answers

(a) The change in the cart's momentum is -200 kg m/s.

(b) The average force that the wall exerts on the cart is 1333.33 N.

(c) The impulse that acted on the cart is in the opposite direction to the cart's initial momentum.

(a) The change in momentum can be calculated as the final momentum minus the initial momentum. The initial momentum of the cart is zero since it was at rest, and the final momentum is calculated as (mass of cart) x (final velocity) = 100 kg x 2 m/s = 200 kg m/s. Therefore, the change in momentum is -200 kg m/s.

(b) The average force can be calculated using the impulse-momentum theorem, which states that the impulse acting on an object is equal to the change in its momentum. The impulse is calculated as (mass of cart) x (final velocity - initial velocity) = 100 kg x (2 m/s - 0 m/s) = 200 kg m/s.

The time taken for the cart to come to a stop is given as 0.15 s. Therefore, the average force exerted by the wall is 1333.33 N.

(c) The direction of the impulse is opposite to the initial momentum of the cart, which was in the direction of the hill. Since the cart was moving downhill, the impulse that acted on it was in the upward direction.

For more questions like Momentum click the link below:

https://brainly.com/question/30677308

#SPJ11

The area of a rectangle park is 53 square mile. The length of the park is 87 mile. What is the width of the park? URGENTTT PLS ANSWER STEP BY STEP

Answers

We can use the formula for the area of a rectangle to solve for the width:

Area = length x width

Substituting the given values, we get:

53 = 87 x width

To solve for the width, we can divide both sides by 87:

53/87 = width

Simplifying, we get:

width ≈ 0.609

Therefore, the width of the park is approximately 0.609 miles.

Rosita is writing an explicit function for the geometric sequence:
80, 40, 20, 10, \dots80,40,20,10,…80, comma, 40, comma, 20, comma, 10, comma, dots
she comes up with t(n)=160\left( \dfrac12 \right)^nt(n)=160(
2
1

)
n
t, left parenthesis, n, right parenthesis, equals, 160, left parenthesis, start fraction, 1, divided by, 2, end fraction, right parenthesis, start superscript, n, end superscript.
what domain should rosita use for ttt so it generates the sequence?

Answers

The domain of the function is the set of all positive integers, since the sequence starts with the first term and continues indefinitely. Therefore, Rosita should use the domain of positive integers for her function to generate the given sequence.

An explicit function is a mathematical expression that directly relates an independent variable to a dependent variable. In the case of Rosita's function, t(n) represents the nth term in the geometric sequence and is dependent on the value of n, the term number.

The explicit function that Rosita came up with is t(n)=160(1/2)^n, which can be simplified to t(n)=80(1/2)^(n-1). This function represents the relationship between the term number and the corresponding value in the sequence.

To determine the domain of the function, we need to consider the values of n that generate the given sequence. Looking at the sequence, we can see that the first term is 80 and each subsequent term is half of the previous term. This means that the sequence is generated by multiplying 80 by (1/2) raised to a power. We can write this as:

80(1/2)^(n-1)

where n is the term number. The domain of the function is the set of all positive integers, since the sequence starts with the first term and continues indefinitely. Therefore, Rosita should use the domain of positive integers for her function to generate the given sequence.

To know more about positive integers, visit:

https://brainly.com/question/18380011#

#SPJ11

What is the perimeter of the triangle?

?? units.

Answers

Answer:

  36 units

Step-by-step explanation:

You want to know the perimeter of the right triangle with leg lengths 9 units and 12 units.

Perimeter

The perimeter of the triangle is the sum of the lengths of its sides. We can count squares to find the lengths of the horizontal and vertical legs. The length of the hypotenuse can be found using the Pythagorean theorem:

  c² = a² +b²

  c² = 9² +12² = 81 +144 = 225

  c = √225 = 15 . . . . length of the hypotenuse

Then the perimeter is ...

  P = a +b +c = 9 +12 +15 = 36 . . . units

The perimeter of the triangle is 36 units.

__

Additional comment

The leg lengths have the ratio 9:12 = 3:4, telling you this is a 3:4:5 right triangle. This means you know the side lengths are 3:4:5 = 9:12:15, and their sum is 9+12+15 = 36.

It is handy to memorize a few of the Pythagorean triples that often show up in algebra, trig, and geometry problems: {3, 4, 5}, {5, 12, 13}, {7, 24, 25}, {8, 15, 17}.

<95141404393>

(65 points) ASAP!!! Warren is building shelves for his 3-D printed model collection. He has a piece of wood that is 4.5 feet long. After cutting five equal pieces of wood from it, he has 0.7 feet of wood left over.

Part A: Write an equation that could be used to determine the length of each of the five pieces of wood he cut. (1 point)

Part B: Explain how you know the equation from Part A is correct. (1 point)

Part C: Solve the equation from Part A. Show every step of your work. (2 points)

Answers

Answer:

Total wood = 4.5 Left over wood = 0.7 let 1 piece of wood = x

hence we have 5x+0.7=4.5

5x=4.5 - 0.7

5x = 3.8 x= 3.8/5 = 0.76 ft

Step-by-step explanation:

Answer:

A: 5x + 0.7 = 4.5

B: You know the equation is correct because it takes into account that Warren cut five pieces of wood that are equal in length from a 4.5 feet long piece of wood. The total length of the five pieces of wood should equal the length of the original piece of wood, minus the leftover wood he has. In other words, 5x represents the total length of the five pieces of wood cut, and 0.7 represents the amount of wood left over after the cutting.

C: 0.76 feet

Step-by-step explanation:

Part A:

Let x be the length of each of the five pieces of wood cut from the 4.5 feet long piece of wood. The equation that could be used to determine the length of each piece is:

5x + 0.7 = 4.5

Part B:

You know the equation is correct because it takes into account that Warren cut five pieces of wood that are equal in length from a 4.5 feet long piece of wood. The total length of the five pieces of wood should equal the length of the original piece of wood, minus the leftover wood he has. In other words, 5x represents the total length of the five pieces of wood cut, and 0.7 represents the amount of wood left over after the cutting.

Part C:

5x + 0.7 = 4.5

5x = 4.5 - 0.7

5x = 3.8

x = 3.8/5

x = 0.76

Therefore, each of the five pieces of wood that Warren cut is 0.76 feet long.

What is the recursive formula for the sequence -1, -3, -9, -33 ...

Answers

The recursive formula for an, the nth term of the sequence is a(n) = a(n - 1) * 2 where a(1) = -1

How to determine the recursive formula of the sequence

From the question, we have the following parameters that can be used in our computation:

-1, -3, -9, -3³ ...

The above definitions imply that we simply multiply 3 to the previous term to get the current term

Using the above as a guide,

So, we have the following representation

a(n) = a(n - 1) * 3

Where

a(1) = -1

Hence, the sequence is a(n) = a(n - 1) * 2 where a(1) = -1

Read more about sequence at

brainly.com/question/29431864

#SPJ1

A company estimates that 0. 5% of their products will fail after the original warranty period but within 2 years of the purchase, with a replacement cost of $500. 0. If they offer a 2 year extended warranty for $33. 00, what is the company's expected value of each warranty sold? Please format your answer using a dollar sign and two decimal points for the cents. For example, if the expected value was 50, please enter it as $50. 0

Answers

The expected value of each warranty sold is $2.47.

The expected value of each warranty sold can be calculated by subtracting the cost of the warranty from the expected value of the potential savings on replacement costs.

Let's assume that the company sells 1000 warranties. Then, the expected number of products that will fail within 2 years is:

0.5% of 1000 = 0.005 x 1000 = 5

The expected cost of replacing these products is:

5 x $500 = $2500

Therefore, the expected value of the warranty is:

($2500 - $33 x 1000) / 1000 = $2.47

To know more about expected value, refer to the link below:

https://brainly.com/question/13479504#

#SPJ11

solve for x:
6x+26=16x

Answers

Step-by-step explanation:

[tex]6x + 26 = 16x\\ 16x - 6x = 26 \\ 10x = 26 \\ x = 2.6[/tex]

Payton bought a 15-year treasury bond for a face amount of $700. The 2. 5% interest will be


compounded quarterly. What will the future value of Patrick's investment be when he goes to


cash it in on the maturity date 15 years from now?

Answers

The future value of Payton's investment will be $1,048.29 when he cashes in the bond on the maturity date 15 years from now.

To calculate the future value of Payton's 15-year treasury bond, we can use the formula for compound interest:

FV = PV * (1 + r/n)^(n*t)

where FV is the future value, PV is the present value (or face amount), r is the interest rate (as a decimal), n is the number of times the interest is compounded per year, and t is the time period in years.

In this case, the present value is $700, the interest rate is 2.5% or 0.025, the interest is compounded quarterly, so n = 4, and the time period is 15 years.

Plugging in the values, we get:

FV = $700 * (1 + 0.025/4)^(4*15)

FV = $700 * (1 + 0.00625)^60

FV = $700 * 1.49756

FV = $1,048.29

Therefore, the future value of Payton's investment will be $1,048.29 when he cashes in the bond on the maturity date 15 years from now.

To learn more about investment, click here:

https://brainly.com/question/15353704

#SPJ11

Ms. Griffin has 0. 8 liters of hot tea and 4 teacups. She will divide the tea evenly among the cups. Which model represents 0. 8 divide by 4

Answers

The model that represents Ms. Griffin's situation is 0.8 divided by 4, which equals 0.2 liters of hot tea in each teacup.

To find the amount of tea in each teacup, you need to divide the total amount of tea (0.8 liters) by the number of teacups (4). The model for this is 0.8 ÷ 4. Follow these steps:

1. Divide 0.8 by 4:
0.8 ÷ 4 = 0.2

2. Interpret the result:
Each teacup will have 0.2 liters of hot tea.

So, the model that represents Ms. Griffin's situation is 0.8 divided by 4, which equals 0.2 liters of hot tea in each teacup.

To know more about "Divide" refer here:

https://brainly.com/question/13753299#

#SPJ11

Need help on the stretch part URGENT

Answers

The equation of the quadratic function in the stretch part is f(x) = x² + 4x - 11

Calculating the equation of the function (the stretch part)

From the question, we have the following parameters that can be used in our computation:

Zeros: -2 ± √15

This means that

Zeros: -2 - √15 and -2 + √15

The equation of the function is calculated as

f(x) = product of (x - zeros)

So, we have

f(x) = (x - (-2 -√15)) * (x - (-2 + √15))

When expanded, we have

f(x) = (x + 2 + √15)) * (x + 2 - √15))

Evaluate the products

f(x) = x² + 4x - 11

Hence, the function is f(x) = x² + 4x - 11

Read more about quadratic functions at

https://brainly.com/question/25841119

#SPJ1

A bee flies for 4.0 min at 32.5 in/min find the bees distance in ft

Answers

The distance that the bees cover in feet is 10.84 feet.

The speed at which the bees travel is given in the unit inches per min but the required solution is in feet so we need to convert the unit from in/min to ft/min using the unit conversion method.

We know that

1 inch=1/12feet.

so 32 inches/min=32.5 *(1/12) feet/min.

which is roughly equal to 2.71 feet/min (rounded to two decimal places).

Now by using the speed, distance, and time formula which is:

distance=speed*time

we can calculate the distance covered by bees at the given speed and time.

Substituting the values in the equation.

distance=2.71 feet/minute * 4.0 minutes.

=10.84 feet

Therefore, the bee's distance in feet will be 10.84 feet (rounded off to 2 digits).

Learn more about the Unit Conversion Method:

https://brainly.com/question/97386

#SPJ4

The value of y varies directly with x. When y = 75, x=1/2. What is the value of y, when x is 2 1/2

Answers

Answer:

y=60

Step-by-step explanation:

y=kx75=k×1/275=1/2k75÷1/2=1/2k÷1/2k=150y=kxy=150×2 1/2y= 60

You suspect that an unscrupulous employee at a casino has tampered with a die; that is, he is using a loaded die. In order to test your suspicion, you rolled the die in question 200 times and obtained the following frequencies for each of the six possible outcomes of the die:


Number Frequency 1 2 3 4 5 6 45 39 35 25 27 29


Can you conclude that the die is loaded? Use a 0. 05 as the significance level and perform a hypothesis test. Remember to state the null and alternative hypothesis

Answers

Based on the hypothesis test, with a significance level of 0.05, there is no evidence to suggest that the die is loaded, as the p-value is greater than the significance level. The null hypothesis that the die is fair is failed to rejected.

To determine if the die is loaded, we need to perform a hypothesis test.

Null Hypothesis (H0) The die is fair; all outcomes are equally likely.

Alternative Hypothesis (Ha) The die is loaded, and not all outcomes are equally likely.

We will use a significance level of 0.05.

To test the hypothesis, we can use a chi-square goodness-of-fit test.

First, we need to calculate the expected frequencies for each outcome, assuming that the die is fair. Since there are six possible outcomes, each with an expected frequency of 200/6 = 33.33.

Number Observed Frequency (O) Expected Frequency (E) (O - E)² / E

1 45 33.33 3.48

2 39 33.33 0.87

3 35 33.33 0.07

4 25 33.33 1.83

5 27 33.33 0.99

6 29 33.33 0.44

The test statistic is the sum of (O-E)² / E, which is 7.68.

The degrees of freedom for this test are (number of categories - 1) = 5.

Using a chi-square distribution table or calculator, we find that the p-value associated with a test statistic of 7.68 and 5 degrees of freedom is approximately 0.177.

Since the p-value is greater than our significance level of 0.05, we fail to reject the null hypothesis. We cannot conclude that the die is loaded based on this data alone.

To know more about Null hypothesis:

https://brainly.com/question/28920252

#SPJ4

solve this problem:

Suppose that you are headed toward a plateau 50 m high. If the angle of elevation to the top of the plateau is 20 ​, how far are you from the base of the​ plateau?

Answers

Answer:

Step-by-step explanation:

We can use trigonometry to solve this problem. Let's call the distance from the base of the plateau to our position "x". We can then use the tangent function to find x:

tan(20°) = opposite / adjacent

In this case, the opposite side is the height of the plateau (50 m) and the adjacent side is x. So we can write:

tan(20°) = 50 / x

To solve for x, we can rearrange this equation:

x = 50 / tan(20°)

Using a calculator, we get:

x = 143.45 meters (rounded to two decimal places)

Therefore, if the angle of elevation to the top of the plateau is 20 degrees, and the plateau is 50 meters high, we are approximately 143.45 meters away from the base of the plateau.

Answer:

The distance is 137.3739 feet.

Step-by-step explanation:

I hope this answer is right.

What is the probability that a student took AP Chemistry, given they did not get into their first-choice college? Enter
your answer as a decimal to the ten thousandths place.
Student
0.25
0.10
0.35
0.30
Chemistry
000
Physics
Env Sci
Biology
0.45
1st choice
Not 1st
0.55
0.55 1st choice
0.45
0.20
0.80
0.30
0.70
Not 1st
1st choice
Not 1st
1st choice
Not 1st
P(Chem and 1st choice) = (0 25)(0.45) = 0.1125
P(Chem and Not 1st) = (0.25)(0.55) = 0.1375
P(Phys and 1st choice) = (0.35)(0.55) = 0.1925
P(Phys and Not 1st) = (0.35)(0.45) = 0.1575
P(ES and 1st Choice) = (0.30)(0.20) = 0.0600
P(ES and Not 1st) = (0.30)(0.80) = 0.2400
P(Bio and 1st choice) = (0.10)(0.30) = 0.0300
P(Bio and Not 1st) = (0.10)(0.70) = 0.0700

Answers

The probability that a student took AP Chemistry given they did not get into their first-choice college is 0.0566.

What is the probability?

The probability that a student took AP Chemistry given they did not get into their first-choice college is calculated using the formula below:

P(Chem | Not 1st) = P(Not 1st | Chem) * P(Chem) / P(Not 1st)

P(Not 1st | Chem) =0.1375

P(Chem) = 0.25

P(Not 1st) = P(Chem and Not 1st) + P(Phys and Not 1st) + P(ES and Not 1st) + P(Bio and Not 1st)

P(Not 1st)= 0.1375 + 0.1575 + 0.2400 + 0.0700

P(Not 1st)= 0.6050

Substituting the values in the formula above:

P(Chem | Not 1st) = 0.1375 * 0.25 / 0.6050

P(Chem | Not 1st) = 0.0566

Learn more about probability at: https://brainly.com/question/25870256

#SPJ1

On the set of axes below, solve the following system of equations graphically and state the coordinates of all points in the solution set.

Answers

The solution to the system of equations shown above is the ordered pairs [-2, -9] and [3, -4].

How to graphically solve this system of equations?

In order to graph the solution to the given system of equations on a coordinate plane, we would use an online graphing calculator to plot the given system of equations and then take note of the point of intersection;

y = -x² + 2x - 1   ......equation 1.

2x - 2y = 14 ......equation 2.

Based on the graph shown in the image attached above, we can logically deduce that the solution to this system of equations is the point of intersection of the lines on the graph representing each of them, which is given by the ordered pairs (-2, -9) and (3, -4).

Read more on solution and equation here: brainly.com/question/25858757

#SPJ1

Suppose a friend or family asked you how it could be possible that an annual interest rate is higher than 100%. write out an explanation of what you might say to them

Answers

If someone asked me how it could be possible for an annual interest rate to be higher than 100%, I would explain that it is actually quite common in certain situations, particularly in the case of loans with very short terms or loans with high fees.

For example, let's say you borrowed $100 from a lender and agreed to pay back $110 in one week. The lender is essentially charging you 10% interest for the one-week loan period, but if you annualize that rate, it comes out to over 520%. This is because the lender is charging you a very high interest rate for a very short period of time.

Another example would be if you took out a payday loan, which typically have very high fees attached to them. For instance, you might borrow $500 and have to pay back $575 in two weeks. The interest rate on this loan might be calculated as the $75 fee divided by the $500 borrowed, which comes out to 15%. However, if you annualize that rate, it comes out to over 390%.

In both of these examples, the interest rate is very high because the loan term is very short and/or the fees are very high. It's important to note that borrowing at such high interest rates can be extremely costly and can lead to a cycle of debt, so it's generally recommended to avoid loans with high interest rates whenever possible.

To know more about " Interest rate" refer here:

https://brainly.com/question/31513017#

SPJ11

A piece of wire of length 50 is out, and the resulting two pieces are formed to make a corde and a square. Where should the wre be cut to day minance and provimine the continet water who? (e) To minimize the combined area, the wire should be cut so that a length of 25.964 used for the circle and a longen er 3.04 es lo quem (Round to the nearest thousandth as needed) (1) To maximize the combined uros, there should be cut so that a length of used for the circle and we canned tere dere (Round to the nearest thousandth as needed) Evaluate the following limit. Use Thôpitals Rule when it is convenient and applicable Iim cox How should the given timt be evaluated? Select the correct choice below and, if necessary, in the answer box to complete your choice A. U topitals Rule more than once to rewrite the imtin ta final fomas tim 9. Multiply the expension by a una traction to obtain im (1) OG UTHopitals Rule exactly once to rewrite the imit im OD. Vse direction Evaluate the limit imetype an exact answer

Answers

To minimize the combined area of a circle and a square made from a wire of length 50, you should cut the wire so that 25.964 units are used for the circle (as the circumference) and 24.036 units are used for the square (as the perimeter).

To maximize the combined areas, the optimal cutting point cannot be determined due to the lack of information provided in the question. For the limit evaluation, it's not clear which limit should be evaluated, as the question has some typos and irrelevant parts. If you can provide the correct limit expression, I will be happy to help you evaluate it using the appropriate method, such as Hsopital's Rule or other techniques.

Learn more about area here:

brainly.com/question/13141049

#SPJ11

x+y=112
y=x-58
using elimination

PLEASE HELP ME!!!!

Answers

Answer:

x=85

y=27

;)

Step-by-step explanation:

x+y=112

y=x-58

add 58 to the other side

58+y=x

Subtract y

x-y=58

x+y=112

Now if we add these we get

2x=170

x=85

Then if we substitute 85 in x+y=112

85+y=112

112

-85

____

27

Check your Answer on

y=x-58

27=85-58

27=27

This is the Answer

Please DM me if I should reexplain THANK YOU!

Hope this helps!

so we have to write the equations in standard form.

x+y=112 is good

let's fix y=x-58

to write it in standard form we have to subtract x from the other side

x-y=58

now we can combine them

x + y = 112
x - y = 58

now you just add going down

2x= 170

170/2x = x = 40.

i think 40 is the answer

Marcus charges 130$ per week to pet sit. Next week he is offering an 18% discount. What is the amount of the discount?

Answers

Answer:23.4

Step-by-step explanation: 18 divided by 130

The amount of the discount is $23.40.

The problem asks us to find the amount of the discount given a percentage discount on a known price. To do this, we use the formula for calculating a percentage of a number.

To calculate the discount amount, we need to find 18% of the original price, which is $130 per week.

We can start by calculating 18% of $130:

Discount = 0.18 * $130 = $23.40

Therefore, the amount of the discount is $23.40.

Other Questions
Wanda's front porch is 7 feet wide and 13 feet long. Wanda wants to stain the wood on the porch next weekend. The stain costs $2. 00 per square foot. How much will it cost to buy enough stain for the whole porch? Please help solve 5 and 6 and show work please Full Term O Question 10 9 pts 5 1 Let f(x) = 3 + 6x? - 153 +3. 2" (a) Compute the first derivative of '(x) = 70 hents (c) On what interval is increasing? interval of increasing = (-2,-5) U (1,60) (d) On what interval is f decreasing? interval of decreasing = (-5,1) **Show work, in detail, on the scrap paper to receive full credit. (b) Compute the second derivative off f''(x) = (e) On what interval is f concave downward? interval of downward concavity = (f) On what interval is f concave upward? interval of upward concavity = **Show work, in detail, on the scrap paper to receive full credit. Write a three- to five-page report explaining the process.explain the agency and type of information you want to see and why. write down reasons that the information might not be available.conduct a search to see if the information is already available. document your search.if the information is not yet available, submit a foia request to that agency. document this process.analyze the processwhat worked, what didnt work, why you think that is, etc.if you receive the information (because it is already available or the government sends it to you) explain if the records you are seeing meet your expectations and why or why not.if you have received the information and it is available online, provide a link. if not, provide a brief description of it. if the information was not yet available and you had to request it, include a copy of your request in the final report. Answer the following questions in complete sentences, and justify your responses.After how many time intervals (shakes) did one-half of your atoms (candies) decay?What is the half-life of your substance?If the half-life model decayed perfectly, how many atoms would be remaining (not decayed) after 12 seconds?If you increased the initial number of atoms (candies) to 300, would the overall shape of the graph be altered? Explain your answer.Go back to your data table and for each three-second interval, divide the number of candies decayed by the number previously remaining and multiply by 100. Show your work.The above percentage calculation will help you compare the decay modeled in this experiment to the half-life decay of a radioactive element. Did this activity perfectly model the concept of half-life? If not, was it close?Compare how well this activity modeled the half-life of a radioactive element. Did the activity model half-life better over the first 12 seconds (four decays) or during the last 12 seconds of the experiment? If you see any difference in the effectiveness of this half-life model over time, what do you think is the reason for it? Six spaceships with rest lengths L0 zoom past an intergalactic speed trap. The officer on duty records the speed of each ship, v. (No ship is going in excess of the stated speed limit of c , so she doesnt have to pull anyone over for a ticket. ) Select the correct answera mine extracts 2 metric tons of coal in an hour. thenumber of hours spent mning, which expression reoa. the expression is at. the amount of oreob. the expression help with this word problem What are guidelines for adding emphasis in documents? Use the given circumference to find the surface area of the spherical object.a pincushion with c = 18 cm Levi finds a skateboard that sells for 139. 99. The store charges 6% sales taxes. About how much money will he have to spend for his skateboard 5. How did the Dutch and British influence Enlightenment thought?6. What opinion did Enlightenment thinkers have about slavery?7. What views did Enlightenment thinkers have about progress? How did that affect their views of different societies?8. How did Enlightenment thought impact production and distribution?9. How did the Enlightenment help or hurt working-class people? For each system of linear equations shown below, classify the system as "consistent dependent," "consistent independent," or "inconsistent." Then, choose the best description of its solution. If the system has exactly one solution, give its solution. (see attached) jameson plans to create a larger kennel by doubling the dimensions in the blueprint. how many times the perimeter of the original kennel is theperimeter of the larger kennel? What is the molar solubility of Ba3(PO4)2. Ksp Ba3(PO4)2 = 1. 3x10-29 Pls help ASAP for school 009 10.0 points Let f be a function defined on (-1, 1] such that f(-1) = f(1) = . Consider the following properties that f might have: A. f(1) = 2; x | | B. f continuous on (-1, 1]; C. Which properties ensure that there exists cin (-1, 1) at which f'(c) = 0? - f(x) = 22/3 = x2 1. B and C only 2. none of them 3. all of them 4. B only 5. C only 6. A and C only 7. A only 8. A and B only A company XYZ is selling good to company ABC at different rates, initially the price was $4 for per unit, and it increased to $4. 5. With the changes in price the quantity demanded by ABC decreased from 420 million tons to 400 million tons. Is the demand elastic? Explain A method for determining whether a critical point is a relative minimum or maximum using concavity. The simplest form of log5+log6-log2